LSAT and Law School Admissions Forum

Get expert LSAT preparation and law school admissions advice from PowerScore Test Preparation.

User avatar
 Dave Killoran
PowerScore Staff
  • PowerScore Staff
  • Posts: 5852
  • Joined: Mar 25, 2011
|
#19174
This post is in response to:

Image


Ok, let's break this down. The problem here occurred because you misinterpreted the last rule by making a Mistaken Reversal, and that is leading you to incorrectly consider answer choice (A). Let’s take a moment to analyze the last rule:

..... "R is not scheduled for Thursday unless L is scheduled for Monday."

The critical part of the sentence is the word "unless," and when "unless" appears it introduces the necessary condition, and then the rest is negated and becomes the sufficient condition (this process is what we call the Unless Equation, which is covered below). Hence, the proper diagram for the rule is:

..... ..... ..... ..... R Th :arrow: L M

Let’s review that again. To diagram a statement involving “unless” (or its analogues “except,“ “until,” or “without”), take the following two steps:

1. The clause modified by “unless” becomes the necessary condition.

2. The remainder is negated—which normally involves removing or adding a “not”—and becomes the sufficient condition.

Thus, in the rule under consideration, “unless” modifies “L is scheduled for Monday” and that becomes the necessary condition; the remainder, “R is not scheduled for Thursday,” is negated to “R is scheduled for Thursday” and becomes the sufficient condition. Hence the diagram: R Th :arrow: L M

So, with this diagram in mind for question 12.1, when L is scheduled for Monday, the necessary condition is met. This does NOT mean, however, that R must be scheduled for Thursday (it is possible but not certain). To infer R is on Thursday would be a reversal of the statement and erroneous. Since (A) tries to say that it must be true that R is on Thursday, answer choice (A) is therefore wrong.


Please let me know if that helps. Thanks!
User avatar
 Dania_ha
  • Posts: 8
  • Joined: May 25, 2021
|
#87331
Hi there!

I was working on the above mentioned question (page 111), and I was able to narrow down my answer choices to either D or E. It seems to me that both answers are correct, because if L were placed on Monday, then none of the other variables could be placed on Monday as well. The answer key however shows answer choice E is not necessarily true, could you elaborate as to why this is the case?

Thank you!

Best,
Dania
User avatar
 Dave Killoran
PowerScore Staff
  • PowerScore Staff
  • Posts: 5852
  • Joined: Mar 25, 2011
|
#87421
Hi Dania,

For starters here, I'm going to refer you to a reply I wrote on this question: viewtopic.php?f=1347&t=7469 [admin note: posts combined - see the first comment by Dave on this thread]

Also, you said the following:
Dania_ha wrote: Tue May 25, 2021 1:12 am It seems to me that both answers are correct, because if L were placed on Monday, then none of the other variables could be placed on Monday as well.
In this setup, it's actually possible for more than one patient to go on a day, so L being on Monday does not close Monday off to other variables.

I think that may help clear this up, but if not just let us know. Thanks!
User avatar
 Dania_ha
  • Posts: 8
  • Joined: May 25, 2021
|
#87533
Oh yes I can’t believe I missed that, thank you! :)

Get the most out of your LSAT Prep Plus subscription.

Analyze and track your performance with our Testing and Analytics Package.